Welcome to the first ever Brilliant Geometry contest.This contest aims to improve the brilliant user's ability to solve olympiad level Geometry problems.
This contest was originally held by ayush rai but he decided to hand the leadership over to me.
The rules of the contest are:
I will post the first problem. If someone solves it, he or she can post a solution and then must post a new problem.
A solution must be posted below the thread of the problem. Then, the solver must post a new problem as a separate thread.
Please make a substantial comment.
Make sure you know how to solve your own problem before posting it, in case no one else is able to solve it within 36 hours. Then, you must post the solution and you have the right to post a new problem.
If the one who solves the last problem does not post a new problem in 24 hours, the creator of the previous problem has the right to post another problem.
It is NOT compulsory to post original problems. But make sure it has not been posted on Brilliant.
If a diagram is involved in your problem please make sure it is drawn by a computer program.
Format your solution in , no picture solution will be accepted. Also make sure your solution is detailed and make sure to proof all claims.
Format your proofs as
SOLUTION TO PROBLEM n
proof here
Question as
PROBLEM n
ask question relevant question here
To answer the latest question just shift to the "newest" mode
Easy Math Editor
This discussion board is a place to discuss our Daily Challenges and the math and science related to those challenges. Explanations are more than just a solution — they should explain the steps and thinking strategies that you used to obtain the solution. Comments should further the discussion of math and science.
When posting on Brilliant:
*italics*
or_italics_
**bold**
or__bold__
paragraph 1
paragraph 2
[example link](https://brilliant.org)
> This is a quote
\(
...\)
or\[
...\]
to ensure proper formatting.2 \times 3
2^{34}
a_{i-1}
\frac{2}{3}
\sqrt{2}
\sum_{i=1}^3
\sin \theta
\boxed{123}
Comments
Problem 4
Let ABC be an acute triangle with D,E,F the feet of the altitudes lying on BC,CA,AB respectively. One of the intersection points of the line EF and the circumcircle is P. The lines BP and DF meet at point Q. Prove that AP=AQ. (IMO Shortlisted)
Log in to reply
Let the two intersection points of EF with the circumcircle be P and P′, with P closer to F than E.
Case 1: The question refers to point P.
Note that BCEF and ACDF are cyclic quadrilateral since the angles at D, E and F are right angles. We will now perform an angle chase. Let ∠BAC=α, ∠ABC=β and ∠BCA=γ. Thus, we have
∠BDF=∠FAC=α∠BFD=∠ACD=γ∠AFE=∠BCE=γ
Since ∠BPA=180∘−∠ACB=180∘−γ, ∠PBA<∠ACB=γ. From this, we gather ∠PBD+∠BDF=∠PBA+∠ABD+∠BDF<α+β+γ=180∘. This implies that Q is on the extensions of BP and DF. This statement is required so as to prove I was not diagram dependent.
Because APBC is a cyclic quadrilateral, ∠QPA=γ. But we also have ∠AFE=γ! Thus, AQPF is cyclic. Since ∠PFB=∠AFE=γ, we must have ∠PQA=γ=∠QPA. Thus, ΔAPQ is isosceles, so AP=AQ.
Case 2: The question refers to P′.
Let the intersection of BP′ and DF be Q′. Using a similar argument as above, we prove I was not diagram dependent and that Q′ is on the interiors of DF and BP′. We also have that since AP′CB is cyclic, ∠AP′Q′=γ. We also have that ∠BFD=γ, so P′AFQ′ is cyclic. Since ∠AFP′=γ, we must have ∠AQ′P′=γ. Thus, ΔAP′Q′ is isosceles. Therefore, AP′=AQ′.
Log in to reply
Great solution. No doubt you were in IMO training!
This works, tho I dont think you needed the extra effort to prove that P's dependancy, it would have been much easier to do separate cases for the proof since both cases are independant themselves.
A much clever way is to use directed angles to even skip the second case!
Log in to reply
PROBLEM 1
Let X and Y be points inside equilateral triangle ABC. Let Y′ be the reflection of Y in line BC. Prove that
XY+XB+XC≥Y′A
note: this problem was originally made by @Sharky Kesa in ayush's thread, i decided to start the contest with this, all credit goes to him
prince loomba posted his solution first but since his solution was wrong the credit goes to sharky kesa, he might post the next problem
Log in to reply
Let X′ be the reflection of X over BC.
If XY+XB+XC<AY′,
⟹X′Y′+X′B+X′C<AY′⟹X′B+X′C<AY′−X′Y′AY′−X′Y′≥0⟹X′B+X′C<0
This is clearly untrue. Thus, we have a contradiction. Therefore XY+XB+XC≥Y′A.
Solution to problem 1
We have to prove that LHS can be greater than RHS, let's take X=centroid of triangle than XA=XB=XC and Y be at the furthest point from A which is B and C. So XY+XB+XC=3XA=3XB=3XC. and AY′=AY. Clearly 3XA is greater than YA because Y is not close to being 3 times as far from X. SO, XY+XB+XC>Y′A.
Combining two equations we have the result
Proof for this to be the worst case
LHS is understood to be minimum when X is centroid
RHS is maximum when Y=A. It can be proved by taking that YY′ is perpendicular to BC and When Y′ is at maximum distance from A, this means that YY′ is maximum. Maximum value of Y′ can be A.
So by the rule of inequality, if we have to prove f (a)>f (b), we prove that the minimum value of f (a) is greater than the maximum value of f (b), So obviously the result is true.
Log in to reply
OK, you havent considered allcases. You specifically checked centroid, but what about an arbitrary point?
Log in to reply
Log in to reply
X being the centroid, since Y is still variable. This is where you have gone wrong.
The LHS cannot be minimum at justLog in to reply
Log in to reply
X and Y have to be fixed for LHS to be minimum. Then, from here, you say that because of Y's location, the RHS is maximised.
But you didn't state that in the statement! Thus, bothLog in to reply
Log in to reply
??? My solution is incorrect?
PROBLEM 2
Let D, E be points on sides BC, AC respectively of triangle ABC such that DE∣∣AB. Let X be a point outside the triangle such that segments XD and XE intersect side AB at points P, Q respectively. Find the smallest value γ such that we must have
∣PQED∣<γ∣ABC∣
The problem has been solved by michael fuller
Log in to reply
I've got γ=21
Place the triangle base on the x axis with B at the origin and A on the x axis. Let the side length of the triangle be s. Let the line y=a be between C and the x axis - it will cut the triangle at the two points D and E from left to right.
The point X can be in the region below the line AB and anywhere "trapped" inside the region bounded by the lines BC extended and AC extended. To find the smallest value of γ we must find the largest value of PQED and this is maximised as the y ordinate of X tends to −∞, where XD and EQ are perpendicular to AB.
∣PQED∣ must be less than the maximum area of a rectangle inscribed in △ABC. Using the line y=a, ∣PQED∣=a(s−2acot60∘)=as−323a2. Differentiating etc to find the optimal value of a gives a=43s and thus ∣PQED∣=83s=2∣ABC∣.
Therefore ∣ABC∣∣PQED∣<21.
Log in to reply
Neat solution! I'd have preferred not to use trig or co-ordinate geometry but this suffices as a solution. My solution is pretty similar but it goes like this:
Let F and G be the feet of the perpendiculars from D and E respectively on AB. Note that FG must contain points P and Q. We have
∣PQED∣<∣FGED∣≤21∣ABC∣
This is sufficient to prove that γ=21.
Log in to reply
:)
Great, I'll get to work on the next problemNeat Solution!
Next problem?
Problem 5
Let D, E, F be points on side BC, CA, AB respectively of triangle ABC. Prove that
min{∣AEF∣,∣BDF∣,∣CDE∣}≤∣DEF∣
Log in to reply
Here || represent?
Log in to reply
The area of that Triangle
Here too I am using the same method.
First minimise area of DEF. This you will get when suppose D and F coincide with B and E with C. But here the area of triangle FBD will become smaller. So LHS is less.
Then Maximise the minimum value of RHS. It can be 1/4 of the area of ABC At max.
So here the equality holds.
After considering the worst cases, the equation is proved.
Proof
Let f(x)≤g(x) to be proved. If we prove that whenever f (x) attains its max value, then too g (x) is greater and when g (x) is minimum then too it is greater than f (x), our result is proved
Thats what I used here!
Log in to reply
This proof is incorrect. Firstly, you have only considered two specific cases. Furthermore, for the rest of the cases, the area of DEF gains, but so is true for the other three triangles, so you cannot apply this method.
Log in to reply
Log in to reply
Log in to reply
If we prove that g (x) is less than the least value of f (x), and that f (x) is greater than the greatest value of g (x), dont you think this is enough for the proof of f (x)>g (x)?
Log in to reply
f(x)=cos(x) and g(x)=sin(x). Say we claim that f(x)>g(x). We have that the greatest value of f(x) is 1 at x=2nπ, which is greater than g(x). Also, the least value of g(x) is −1 is at x=23π+2nπ, which is less than f(x). Thus, by what you are saying, f(x)>g(x), which is clearly incorrect.
No it isn't. Example isSolution
Here is my proof to this question, since no one has posted one themselves correctly.
Perform an affine transformation to convert ΔDEF into an equilateral triangle. Let X be the point distinct from D such that ΔXEF is also an equilateral triangle. WLOG ∠BAC≥60∘. Since ∠BAC≥∠EXF, A must not exist outside the circumcircle of XEF. Thus, the area of AEF is less than or equal to the area of XEF. But the area of XEF is the same as the area of DEF. Thus, ∣AEF∣≤∣DEF∣. Therefore, min{∣AEF∣,∣BDF∣,∣CDE∣}≤∣DEF∣.
Problem 6
Let ABC be an isosceles triangle, with ∠ABC=∠ACB=80∘. Point P is on AB such that AP=BC. Find ∠PCB with proof.
Log in to reply
Here is one such Euclidean solution:
Construct point X such that X is closer C than B and ΔXAP≡ABC. Angle chasing results in ∠CAX=60∘. Notice that ΔACX is isosceles, but ∠CAX=60∘. Therefore, ΔACX is equilateral. Further angle chasing results in ∠PXC=40∘, and after a few angles, we get ∠PCA=10∘. Therefore, ∠BCP=70∘.
Log in to reply
What was the motivation for the construction?
Log in to reply
Log in to reply
AP=BC is the motivation, since we want to try to relate these unrelated segments in some manner.
The lengthI used trigonometry and got the answer as 70 degrees. Should I post the solution?
Log in to reply
If You have solved it You surely may!
And the answer is correct I guess
Can you find a solution that doesn't use trigonometry? The problem is meant as a challenge to solve via only Euclidean geometry.
Log in to reply
Log in to reply
Problem 7
ABCD is an isosceles trapezium with BC∥AD and ∠CAD=45∘. E is a point on AB such that DE⊥AB. F is a point on DE such that CF⊥DE. Prove that 2∠FBD=∠FCD.
Log in to reply
Hey, This doesnot seem to be true.
However the following are true: EA=EF and that (EFA) is tangent to CF
Log in to reply
Sorry, I forgot one other condition.
Log in to reply
Let BD∩AC=G, Note that with our angle condition, DFGC is cyclic, but also note that △CGD∼△BED⟹ BD bisects ∠CDF⟹ GFC is iscoceles, but also note that GB=GC⟹G is the circumcenter of △BFC
Finishing off we have ∠FCD=∠FGD=2∠BFG
Problem 9
Let two circles Γ1 and Γ2 (with Γ1 being larger than Γ2) be internally tangent at M. A, B and C are points on Γ1, such that AB and AC are tangent to Γ2 at P and Q respectively. Let MP and MQ intersect Γ1 for the second time at X and Y respectively. Prove that the intersection of XC and YB is on the line PQ.
Log in to reply
Consider the hexagram MXCABY, then Pascal's Mystic Hexagram Theorem states that the intersections of the opposite sides: MY and AC (Q), MX and AB (P), and BY and CX (the intersection in question), are collinear.
Log in to reply
Just as an extension, prove this intersection is the incentre of ΔABC.
Log in to reply
AP=AQ
Follows from it lying on the angle-bisector andPROBLEM 10
Consider a right triangle ABC with m∠C=90°. Let D be the foot of the altitude from C. Let E be a point in the on the line segment CD.
Let F be the point on the segment AE such that BF=BC. Similarly, let G be the point on the segment BE such that AG=AC. Let X be the point of intersection of AG and BF.
Show that XF=XG. (IMO Past Question)
Log in to reply
This was an extremely tough question, I thought, but I think I've managed to scrape up a solution, only because a similar, simpler setup was considered during my selection camp for IMO. I would have no clue how to solve this without that. My solution will explore how to prove this through Power of a Point.
Firstly, we will create some more labels. Let the circle with radius AG be Γ1, and the circle with radius BF be Γ2. Also, Let A′ be the intersection of AE and Γ2, B′ be the intersection of BE and Γ1, and C′ be the reflection of C over AB.
Some preliminary investigation leads us to conclude AC is tangent to Γ2 since ∠ACB is 90∘. Similarly, BC is tangent to Γ1.
We will now prove F, G, A′ and B′ are cyclic. Note that C and C′ are the intersections of Γ1 and Γ2, so CC′ is the common chord of Γ1 and Γ2. We have
EF×EA′=EC×EC′=EG×EB′
by Power of a Point. Thus, EF×EA′=EG×EB′, so FGA′B′ is a cyclic quadrilateral. Let the circle circumscribing this be Γ3. We will now prove BF is tangent to Γ3. We have
BFBF2=BC=BC2=BG×BB′
The last statement is from Power of a Point in Γ1. Now, from Power of a Point in Γ3, since BF2=BG×BB′, we have BF is tangent to Γ3. Similarly, AG is tangent to Γ3 as well. Thus, XG and XF are both tangents to Γ3, so XG=XF.
Log in to reply
Beautiful solution,
Which year is this from btw?
Log in to reply
Great job! I can reveal the problem was IMO 2012 Question 5.
Log in to reply
No odffence to Sharky's great solution tho!
PROBLEM 3
Let ABCD be a convex quadrilateral such that diagonals AC and BD intersect at right angles - let E be their intersection.
Prove that the reflections of E across AB, BC, CD, DA are concyclic, i.e. the four reflected points connect to form a cyclic quadrilateral.
This problem has been solved by Sualeh Asif.
Log in to reply
@Michael Fuller this problem is trivial by inversion about E.
Let the Refleted points be Eab,Ebc,Ecd,Eda. Now consider the circles through the points E,Eab,Ead and so on. Call them ΓA,ΓB,ΓC,ΓD. Note that ΓA is centered at A since AE=AEab=AEad. Now apply an inversion about E with arbitrary radius. Note that ΓA,ΓC, are tangent and so are ΓB,ΓD,. Under the inversion A′,B′,C′,D′ are arbitrary such that A′C′⊥B′D′. The critical observation is that ΓA′ is inverted to a line perpendicular to A′C′ since ΓA is perpendicular to AC. Similarly we get two pairs of parallel lines ΓA′,ΓC′ and ΓB′,ΓD′. And hence the points Eab′,Ebc′,Ecd′,Eda′ are just the vertices of a rectangle which is obviously cyclic. Hence Eab,Ebc,Ecd,Eda are also cyclic!
Log in to reply
Can you explain?
Log in to reply
Log in to reply
:)
Yes, this works, I believeMy solution (or rather, the one I found with this problem) goes as follows:
Let X, Y, Z, W be the feet of the altitudes from point E to △AEB, △BEC, △CED, △DEA respectively.
Note that the reflection of E over the 4 lines is XYZW with a scale factor of 2 with center E. Thus, if XYZW is cyclic, then the reflections are also cyclic.
∠EWA is right angle and so is ∠EXA. Thus, EXAW is cyclic with EA being the diameter of the circumcircle.
It follows that ∠EWX≅∠EAX≅∠EAB because they inscribe the same angle. Similarly ∠EWZ≅∠EDC, ∠EYX≅∠EBA, ∠EYZ≅∠ECD.
Futhermore, m∠XYZ+m∠XWZ=m∠EWX+m∠EYX+m∠EYZ+m∠EWZ=360∘−m∠CED−m∠AEB=180∘.
Thus, ∠XYZ and ∠XWZ are supplementary and it follows that XYZW is cyclic.
Problem 8
Let ABC be triangle with incenter I . A point P in the interior of the triangle satisfies ∠PBA+∠PCA=∠PBC+∠PCB . Show that AP≥AI , and that equality holds if and only if P=I.
(IMO shortlisted)
Log in to reply
Let ∠A=α, ∠B=β and ∠C=γ. We will first prove that BIPC is a cyclic quadrilateral. We have:
∠PBA+∠PCA+∠PBC+∠PCB=β+γ, so ∠PBC+∠PCB=2β+γ. Note that this implies ∠BPC=90∘+2α.
Since I is the incentive of ABC, it can be angle chased to be shown that ∠BIC=180∘−2β+γ=90+2α. Therefore, ∠BPC=∠BIC, so BIPC is a cyclic quadrilateral.
By Charles' Lemma, we must have the centre of this cyclic quad is the intersection of the line AI extended with the circumcircle of ABC. Call this point X. We have the following:
AP+PX≥AX=AI+IX=AI+PX⟹AP≥AI
with equality if I and P concur.
Problem 11
Definitions: Let O be the centre of a circle Γ with radius r. Let K be a point. Let K′ be the point on OK (possibly extended) satisfying OK×OK′=r2, with respect to Γ. Let f(K) denote the line perpendicular to K′.
Given a cyclic quadrilateral ABCD. Let AC∩BD=P, AB∩CD=Q, AD∩BC=R. Prove f(P)=QR, f(Q)=PR, f(R)=PQ.
Log in to reply
Hey Sharky, this is a well-known Theorem-Brokard's Theorem:
It claims that PQR is self-polar,(what you stated above), and that as a corollary O is the orthocenter of PQR
Log in to reply
But can you prove it?
Log in to reply
But Ill write the proof in a bit
Note that f(Q)=PR⟹PRis the polar of Q. So now we define a few more points:PR∩CD=E,PR∩AB=F,AC∩QR=G,BD∩QR=H, now just note that by perspecting through P, we have, (Q,R;G,H)=(Q,E;C,D)=(Q,F;A,B)=−1
Which simply implies that EF=PR is the polar of Q. similarly f(R)=PQ⟹PR,
We can now end in several different ways, here is one of them that I like a lot.
1) Note that the two polar relationships imply that O is the orthocenter of PQR, and hence OP⊥QR. But inverting in (O), we have that the inverse of P=AC∩BD is the M, the miquel point of ABCD , which we know by simple angle-chasing that it lies on QR. This combined with OP⊥QR⟹P′=M⟹QR is the polar of P⟹f(P)=QR
Brokard
Problem 12
Here is a problem I loved alot when I was given this, because of its utter simplicity:
Let the nine-point circle of △ABC intersect BOC at X,Y.Prove that ∠BAX=∠CAY
(I guess this is from Serbia)
Log in to reply
Let Mb,Mc be the midpoints of AC,AB, respectively. Let H be the projection of A onto BC and let ℓ be the internal bisector of ∠BAC. Let Ha be the foot of the A-perpendicular.
The transformation T composed of an inversion with center A and power 21⋅AB⋅AC with a reflection in ℓ swaps B,Mb and C,Mc. Moreover, note that T(O)∈AH (because circumcenter and orthocenter are isogonal conjugates) and B,C,T(O) are collinear because A,Mb,Mc,O are concyclic. Hence, T(O)≡Ha. It follows that T swaps ⊙(MbHaMc) and ⊙(BOC). Then since ⊙(MbHaMc) is just the nine-point circle of △ABC, the intersections X,Y of the nine-point circle and ⊙(BOC) are swapped by T. Hence, AX,AY are isogonal WRT ℓ, as desired.
(Solution by Dukejukem on AoPS.)
Challenge: There is a nice solution using similar triangles and angle-chasing, Find it.
Hint: Construct O,H
Here is the general problem and solution by Xuming Liang:
Let C′∈AB,B′∈AC such that B,C,B′,C′ are concyclic. Suppose ω,ω′ are two circles through B,C and C′,B′ respectively, such that they are corresponding circles between similar triangles ABC,AB′C′. If ω∩ω′=X,Y, prove that AX,AY are isogonals wrt ∠BAC.
Here are two solutions by him:
Serbian Problem
1 week,12 problems.
Problem 13:
X and Y are two points lying on or on the extensions of side BC of △ABC such that XAY=90. Let H be the orthocenter of △ABC. Take X′ and Y′ as the intersection points of (BH,AX) and (CH,AY) respectively. Prove that circumcircle △CYY′,circumcircle of △BXX′ and X′Y′ are concurrent.
Pakistan TST 2016 Problem 4, Iranian 3rd round Geometry exam P5 - 2014